LSAT and Law School Admissions Forum

Get expert LSAT preparation and law school admissions advice from PowerScore Test Preparation.

User avatar
 Dave Killoran
PowerScore Staff
  • PowerScore Staff
  • Posts: 5852
  • Joined: Mar 25, 2011
|
#41279
Complete Question Explanation
(The complete setup for this game can be found here: lsat/viewtopic.php?t=11660)

The correct answer choice is (D)

Note that the question stem asks for a complete and accurate list, and the correct answer must list all of the factories that could appear on day 1. So, the correct answer must contain each and every factory that is possible for day 1. As strange as it may sound, the question asks for what must be true about what is possible.

If G and H are to be scheduled as far apart as possible (that is, the distance between them is maximized), this would likely place G and H in days 1 and 5 or days 2 and 6 (do not forget that F must be in either day 1 or day 6). Again, there are several ways to approach this question depending on your level of game understanding. At the most advanced level, answer choices (A), (C), and (E) can likely be eliminated since each contains J. If J is inspected on day 1, then F would be inspected on day 6 and G and H would not be as far apart as possible—a violation of the “if” clause in the question. Only F differentiates answer choice (B) from answer choice (D), and since F is a day 1 or day 6 player, it seems likely that answer choice (D) is correct, and in fact it is the correct answer. If that type of theoretical analysis makes you a bit nervous, you can always resort to the other form of attack on this question: make a few quick hypotheticals. Here is one that eliminates answer choices (B) and (C): F-G-J-Q-R-H. Another that eliminates answer choice (A) is: G-J-Q-R-H-F. And again, the inclusion of J in answer choice (E) should be a tip-off that answer choice (E) is likely incorrect.
 lakasil
  • Posts: 10
  • Joined: Aug 08, 2016
|
#28522
On page 161-162 in the LGB I have the answer correct, but besides the book explanation why is (d) correct when G and H are right next to each other?
 Nikki Siclunov
PowerScore Staff
  • PowerScore Staff
  • Posts: 1362
  • Joined: Aug 02, 2011
|
#28551
Hi lakasil,

Thanks for your question.

The stem asks us to determine the complete and accurate list of the factories, any one of which could be on day 1. The correct answer choice (D) merely provides us that list, in no particular order (in fact, the order in which the variables are listed is alphabetical). If I understand your question correctly, you're puzzled by the fact that G and H appear to be adjacent in answer choice (D). Since the answer choice merely presents a list of variables, this appearance is purely coincidental (clearly, they need to be as far apart as possible, as the stem indicates).

Hope this helps.

Thanks!
 KG!
  • Posts: 69
  • Joined: May 26, 2020
|
#76737
Administrator wrote:Complete Question Explanation
(The complete setup for this game can be found here: https://forum.powerscore.com/lsat/viewtopic.php?t=11660)

The correct answer choice is (D)

Note that the question stem asks for a complete and accurate list, and the correct answer must list all of the factories that could appear on day 1. So, the correct answer must contain each and every factory that is possible for day 1. As strange as it may sound, the question asks for what must be true about what is possible.

If G and H are to be scheduled as far apart as possible (that is, the distance between them is maximized), this would likely place G and H in days 1 and 5 or days 2 and 6 (do not forget that F must be in either day 1 or day 6). Again, there are several ways to approach this question depending on your level of game understanding. At the most advanced level, answer choices (A), (C), and (E) can likely be eliminated since each contains J. If J is inspected on day 1, then F would be inspected on day 6 and G and H would not be as far apart as possible—a violation of the “if” clause in the question. Only F differentiates answer choice (B) from answer choice (D), and since F is a day 1 or day 6 player, it seems likely that answer choice (D) is correct, and in fact it is the correct answer. If that type of theoretical analysis makes you a bit nervous, you can always resort to the other form of attack on this question: make a few quick hypotheticals. Here is one that eliminates answer choices (B) and (C): F-G-J-Q-R-H. Another that eliminates answer choice (A) is: G-J-Q-R-H-F. And again, the inclusion of J in answer choice (E) should be a tip-off that answer choice (E) is likely incorrect.
I ended up being very lucky on the question and ended up eliminating B and C based off the fact that well F has to be included in the answer choice because F could be Day 1 or Day 6. And then I later saw that A,D,and E all contained G or J so, I ended up focusing on the differences between them. Did I approach this problem entirely wrong? Basically was I wrong to assume because F could be Day 1 or 2 that I could easily eliminate B/C ?

Also, based on your explanation is it a good tactic for narrowing down or even looking at all the answer choices (regardless for time right now and to better understand the approach) seeing how many answer choices include the same variable?

I noticed I have major difficulties in all the games when it comes to Max or Minimum questions. I have the Bible, but was wondering if you have any other additional tips on how to approach this problem?

Thank you so much in advance for your help! Loving the forum!
 Jeremy Press
PowerScore Staff
  • PowerScore Staff
  • Posts: 1000
  • Joined: Jun 12, 2017
|
#76806
Hi KG!

I wouldn't say you got overly lucky, but I would offer just a word of caution. It's true in a general sense that, according to the initial conditions, F must be in day 1 or day 6. But since this is a local question, you do have to stop for a second and ask whether, when you apply the local condition (G and H as far apart as possible), F could still be in day 1 (rather than being forced into day 6)? The good news is that when you apply that local condition, you'll see it doesn't matter whether F is in day 1 or day 6, because, with F at either extreme end of the line, G and H can still be the same distance apart (which is as far apart as they can possibly be). So that just means F could be in day 1, and you're off and running with your way of working through the question.

The best approach to this question (and to all max/min questions) is to start with thinking about what they're asking for, here the maximum. What's the maximum distance G and H could ever be apart? The maximum they could ever be apart (even independent of the rules) is to put them on 1 and 6. Oops! That doesn't work, because I need F to be in one of those slots. So think about what happens when you put F in 1: now G or H shifts to 2, and the other one is still in 6. Think about what happens when you put F in 6: now G or H shifts to 5, and the other one is still in 1. Those two represent the universe of possibilities where G and H are as far apart as possible. And the only things that can be first in that universe of possibilities are F, G, and H. Bingo!

I hope this helps!

Jeremy
 nickp18
  • Posts: 20
  • Joined: May 26, 2020
|
#87576
Hi Powerscore team!

Will you please explain the thought process behind the wording "as far apart as possible"? I have worked on this problem twice, several days apart and made the same mistake in selecting J as a possible variable to be inspected first. How do we know precisely what "as far apart as possible" means in questions similar to this one? Is anything under 3 spaces considered not as far as possible?

Thanks!

Nick
User avatar
 Poonam Agrawal
PowerScore Staff
  • PowerScore Staff
  • Posts: 71
  • Joined: Apr 23, 2021
|
#87600
Hi Nick!

When the question asks us to place G and H as far apart as possible, we have to maximize the amount of spaces between the inspections of G and H. So, you want to start by first figuring out how to maximize the distance between G and H before even starting to test out other variables, like J.

To figure out what the maximum distance can be between G and H, you can start by thinking about the extreme possibility of having G and H be on opposite ends in spots 1 and 6. However, this does not work within the constraints of the game, since that leaves nowhere for F to be placed. Next, you can try moving either G or H one space closer and placing F on the end, instead. This can be done in four ways:

1.) G __ __ __ H F
2.) H __ __ __ G F
3.) F G __ __ __ H
4.) F H __ __ __ G

Note that in each of the four ways above, there are exactly three spots between G and H. Therefore, this is the maximum possible distance between G and H.

The next step is to make sure that the four diagrams above work with the remaining variables, JQR. These three variables can just go consecutively in our three blank spaces in each of the four diagrams with no problems. This confirms that our diagrams are okay to use to answer the question.

The last step is to observe the four diagrams and answer the question of which variables can go in the first spot. This turns out to be G, H, and F, which is answer choice (D).

I hope this clarifies the thought process for this question! Let us know if you have any other questions.
 Tasha68
  • Posts: 3
  • Joined: Jun 20, 2021
|
#89417
Hello,

I am very much struggling with the reasoning which is given in the Bible and I don't think I quite understand what "complete variable list" exactly means.

When the answer choices contain a variable list, example (D) F,G.H, are we suppose to diagram the possibilities with each one of them in Day 1? Like if F in 1 doesn't violate any rule in the game, F is a possibility, and G in 1 won't violate any rule unless F is 6 and so on.

Are we suppose to think of the consequences of each variable in a particular answer choice and evaluate if it violates any rules?

I know I completely got it wrong but I just want to make sure I understand, I initially thought the answer choices, which are like the one under #6, all of the variable separated by a "," are supposed to be together for day 1 but I understand that is not the case, could someone please simplify it so I can know what it truly and actually means?

Thank you so much!
User avatar
 Beatrice Brown
PowerScore Staff
  • PowerScore Staff
  • Posts: 75
  • Joined: Jun 30, 2021
|
#89505
Hi Tasha! Thanks for your question :)

This question is asking us for a complete and accurate list of the factories that are able to be inspected first when G and H are as far apart as possible. There are two types of incorrect answer choices: 1) the answer choice leaves out a variable that can be inspected first, making the list incomplete; and 2) the answer choice includes a variable that cannot be inspected first because one of the rules of the game would be violated, making the list inaccurate.

The variables listed in the answer choices do not have to all be inspected first (since we're dealing with a linear game where only one variable can go in each slot), but it needs to be possible for the variable contained in that list to be inspected first without one of the rules of the game being violated.

Your approach of thinking through the consequences of each variable in that answer choice being selected first and eliminating that answer choice if a rule is violated by that variable being selected first is correct :) For this particular question, you want to examine what happens if one of these variables is in slot 1 but G and H are in 2 and 6 (in either order) or in 1 and 5 (in either order), since these are the situations in which G and H are farthest apart.

I hope this helps, and let me know if you have any further questions!

Get the most out of your LSAT Prep Plus subscription.

Analyze and track your performance with our Testing and Analytics Package.